দুটি বিতরণ বিতরণ যুক্ত করার ফলে কী বিতরণের ফলাফল হয়


12

আমি দুই যুক্ত করতে কি বন্টন ফলাফল হতাশ করছি (বা আরো) ফর্ম ধরণ এক Pareto ডিস্ট্রিবিউশন । পরীক্ষামূলকভাবে, এটি একটি দ্বি-মোড পাওয়ার-ল এর মতো দেখায়, বর্ণমালার পার্থক্যের প্রতিসাদৃশ্য।এক্স-α


3
সর্বশেষ মন্তব্যটি আপনার মতামতটিকে বিতরণের মধ্যে বর্ণমালার সাথে পৃথক হওয়া আলফাসের মতো ভাবনার মতো করে তোলে। আপনি কি বিতরণগুলির ডোমেনগুলি (ওরফে "স্কেল") ঠিক করতে যাচ্ছেন নাকি? একটি দ্রুত গাণিতিক গণনা পিডিএফ এর শর্তগুলির মধ্যে একটি হিসাবে এবং বিটা বিতরণের পার্থক্য এবং বিটা বিতরণ । এটি । এই ফলাফলটি বৃহত্তর এবং ধরে রাখে না , সুতরাং যে প্যারামিটারগুলিতে আপনি আগ্রহী তার সম্ভাব্য মানগুলির কোনও সীমাবদ্ধতা রয়েছে? ( - α , 1 - β ) 1 - 1 / এক্স 1 / এক্স 0 < α < β < 1 α βএক্স-α-β(-α,1-β)1-1/এক্স1/এক্স0<α<β<1αβ
whuber

2
নিম্নলিখিত কাগজটি সিডিএফের সম্প্রসারণ এবং এটি আনুমানিক করার একটি উপায় প্রস্তাব করেছে: ডকসস.আইএসএফ.ফার
labo

উত্তর:


6

কিছুটা বেশি পঠনযোগ্য হতে সম্পাদিত। বিতরণ সংশ্লেষ দ্বারা যোগ করুন। পেরেটো ডিস্ট্রিবিউশনটি কে হিসাবে সংজ্ঞায়িত টুকরা অনুসারে জন্য and এবং জন্য 0 । দুটি পেরেটো ফাংশনগুলি এবং functions এর হ'ল: x k x < k k a x - a - 1 j b x - b - 1একটিএক্স-একটি-1এক্সএক্স<একটিএক্স-একটি-1এক্স--1

একটি(-1)-একটিΓ(একটি+ ++ +1)×((1টি-)একটি+ ++ +12এফ~1(+ +1,একটি+ ++ +1;একটি+ ++ +2;টিটি-)-(1)একটি+ ++ +12এফ~1(+ +1,একটি+ ++ +1;একটি+ ++ +2;টি)),

যেখানে এবং 0 জন্য , যা যদিও এটি যে শব্দ, বাস্তব মূল্যবান হয় বাহিরে মধ্যে জটিল ক্ষেত্র। হ'ল হাইপারজেমেট্রিক 2 1 গণিতের কোডে এখানে নিয়মিত । পরামিতিগুলির জন্য সমস্ত পছন্দ ধনাত্মক মূল্যবান ঘনত্বের কার্যগুলি দেবে না। তারা কখন ইতিবাচক হয় তার উদাহরণ এখানে। দুটি পেরেটো বিতরণের জন্য একটি = 2, বি = 3, জে = 0.1 এবং কে = 0.3 থাকুন। এবং তাদের প্লটগুলি {k, a} ফাংশনের জন্য নীল এবং {j, b} ফাংশনের জন্য কমলাতে। তাদের কনভলিউশনটি তখন গ্রাফিক্যালি যা যা লেজগুলি পরীক্ষা করা হয় দেখে মনে হয় সবুজটি কোথায় কনভোলশন।x জে + কে+ +<এক্সএক্স+ +2এফ~1(W,এক্স;Y;z- র)
কোথায়

এখানে চিত্র বর্ণনা লিখুন

এখানে চিত্র বর্ণনা লিখুন

আপনার প্রশ্ন থেকে, আপনি দুটি পেরেটো বিতরণের সাধারণ সংযোজন সম্পর্কে জিজ্ঞাসা করতে পারেন। সেক্ষেত্রে বক্ররেখার ক্ষেত্রফল দুটি, সুতরাং যোগফলটি একটি ঘনত্বের ফাংশন নয়, যার একটির বক্ররেখার অধীনে একটি অঞ্চল থাকা দরকার। যাইহোক, যে যদি প্রশ্ন তারপর জন্য সহজসাধ্য করার জন্য , যার মধ্যে একটা সীমা আছে শুধুমাত্র যদি , এবং অন্যান্য সব ক্ষেত্রে 0 বা অনন্ত। অন্য কথায়, দুই Pareto ডিস্ট্রিবিউশন এর গাণিতিক সমষ্টি শুধুমাত্র মুদ্রার উলটা পিঠ যে পার্থক্য রয়েছে এবং যখনএকটিএকটিটি-একটি-1+ +টি--1টিএকটি--1>একটি>0টি-2একটি(টিএকটি+ +একটিএকটিটি)একটিএকটি=2একটিএকটি=2একটি, এবং গাণিতিক যোগফল একটি ঘনত্বের ফাংশন নয় এবং ঘনত্বের ক্রিয়াকলাপ হওয়ার জন্য যোগফলটিকে দুটি সম্ভাব্যতার জন্য করতে হবে। অন্য ঘনত্বের ক্রিয়াটি সংজ্ঞায়িত করার জন্য ঘনত্বের ফাংশনগুলির গাণিতিক সংযোজন ঘটে গেলেও এটি অস্বাভাবিক is এর একটি উদাহরণ ফার্মাকোকাইনেটিক্সে দেখা যায়, যেখানে ঘনত্বের ক্রিয়াটি সংজ্ঞায়িত করতে দুই বা ততোধিক ক্ষতিকারক বিতরণের যোগফল ব্যবহৃত হয়। একটি দীর্ঘ গল্প সংক্ষিপ্ত করতে, এটি এমন কিছু নয় যা আমি সুপারিশ করব।1=পি+ +কুই

আশা করি এটি আপনার প্রশ্নের উত্তর দেয়। যদি তা না হয় তবে দয়া করে আমার উত্তরটি আপত্তি করুন বা দয়া করে আরও কিছু তথ্য যুক্ত করুন।


1
@ গুং পরিষ্কার করার জন্য ধন্যবাদ। এর জন্য আমার কি কিছু শিষ্টাচার দরকার? কেউ কি সাফার জন্য সুনাম বরাদ্দ পায়, বা কেবল ভাল ইচ্ছা?
কার্ল 21

1
আপনি স্বাগত, @ কার্ল। যদি আপনার খ্যাতি <2 ক (?) হয়, আপনি যখন কোনও সম্পাদনার প্রস্তাব দেন এবং এটি অনুমোদিত হয়, আপনি +2 পান। এর পরে, সম্পাদনাগুলি আপনাকে কিছুই দেয় না। আমার প্রতিনিধি দরকার নেই তাই সমস্যা নেই no আপনার উত্তরটি এখানে ভাল (+1), এটি পড়ার সামান্য সহজ করার জন্য আমি এটি সম্পাদনা করেছি।
গুং - মনিকা পুনরায়
আমাদের সাইট ব্যবহার করে, আপনি স্বীকার করেছেন যে আপনি আমাদের কুকি নীতি এবং গোপনীয়তা নীতিটি পড়েছেন এবং বুঝতে পেরেছেন ।
Licensed under cc by-sa 3.0 with attribution required.